3
$\begingroup$

A subset $A\subseteq \mathbb{R}$ is said to be a $Q$-set if every subset $B\subseteq A$ is $F_\sigma$ wrt the subspace topology on $A$. For example $\mathbb{Q}$ is a $Q$-set. The first time I have seen this definition is in: Balogh, Zoltán, There is a $Q$-set space in ZFC, Proc. Am. Math. Soc. 113, No. 2, 557-561 (1991).
Now my questions are:

  1. Can we prove in $\mathtt{ZF}+\mathtt{DC}$, where $\mathtt{DC}$ stands for dependent choice, that every co-analytic $Q$-set is countable?
  2. Is the descriptive complexity (with connections with set-theoretic hypothesis) of these sets been studied in any paper/book/thesis?

Thanks!

$\endgroup$

1 Answer 1

7
$\begingroup$

I do not know about full references but if you haven't seen it yet there is a nice discussion of this topic in Miller's https://people.math.wisc.edu/~miller/res/dstfor.pdf. (Incidentally this may be the most fun math book to read). See in particular sections 2-5.

In particular, Theorem 5.1 states that Martin's Axiom implies every second countable Hausdorff space of cardinality less than the continuum is a Q-set. If $V=L$ there are $\Pi^1_1$ sets $A \subseteq \mathbb R$ of size $\aleph_1$ so that in any $\aleph_1$-preserving forcing remain uncountable and no new elements are added. Forcing martin's Axiom over $L$ with ccc forcing (so the standard way) will therefore give a consistent example of an $\aleph_1$-sized co-analytic Q-set.

Also Corollary 3.2 can be used to answer your first question. This corollary states that any second countable space which contains a perfect set is not Q (actually it uses a weaker hypothesis to make a stronger conclusion). Staring at the proof of the relevant results towards proving this corollary you need to convince yourself that DC is enough. Given this, if every set $A \subseteq \mathbb R$ contains a perfect set (for instance in the Solovay model) then no uncountable set of reals can be a Q-set.

Putting these two results together also we get (modulo an inaccessible) that the existence of a projective uncountable Q-set is independent of ZFC.

I do not know if anyone has thought about these definability issues further, though I would check the references of the Miller book to start.

$\endgroup$
4
  • $\begingroup$ Thanks for the answer and for the reference. I have a question which may be a bit silly: once we force Martin's Axiom, how can know that the set $A$ is still $\Pi_1^1$ in the forcing extension? Thanks $\endgroup$
    – Lorenzo
    Apr 23, 2022 at 10:20
  • $\begingroup$ There are two ways of interpreting this question. A $\Pi^1_1$ set is a set defined by a formula of the form $\forall x \in \mathbb R \varphi(x, y)$ with $\varphi$ arithmetic. Of course in any forcing extension this formula still exists and so it re-evaluates to a $\Pi^1_1$ set. However, when we force, two (non mutually exclusive) things can happen - new reals $x \notin V$ can be forced to satisfy this formula and/or the set of x satisfying the formula can become countable. If we force MA with ccc forcing over $L$ then of course the latter does not happen. The point is that in L we can... [1/3] $\endgroup$ Apr 25, 2022 at 12:58
  • $\begingroup$ ...in L we can find a $\Pi^1_1$ set $A$ so that $\mathsf{ZFC} \vdash$``$A \subseteq L \cap \mathbb R$" i.e. it is provable that no non-constructible reals satisfy the defining formula for $A$. There are many examples of such. A particularly obvious one is the set of reals coding an $L_\alpha$ for $\alpha < \omega_1$ (for some fixed coding that makes it constructible). [2/3] $\endgroup$ Apr 25, 2022 at 13:02
  • $\begingroup$ Note that in L none of these sets are Q however - CH implies that there are no uncountable Q sets (in fact $2^{\aleph_0} < 2^{\aleph_1}$ suffices). The point is that for any subset $B$ of any such $A$ there is a ccc forcing to make $B$ relatively $G_\delta$ in $A$ (and this persists upwards) so under MA every such set is relatively $G_\delta$. [3/3] $\endgroup$ Apr 25, 2022 at 13:02

Your Answer

By clicking “Post Your Answer”, you agree to our terms of service and acknowledge you have read our privacy policy.

Not the answer you're looking for? Browse other questions tagged or ask your own question.